Luận văn Chứng minh một số bất đẳng thức cơ bản bằng phương pháp hình học

Trong luân văn na ̣ ̀y tác giả đã đọc, nghiên cứu và viết lại một số các kết quả sau. 1. Thiết lập được bất đẳng thức AM-GM cho hai số dương và một số bất đẳng thức trung bình khác bằng cách sử dụng một trong các phương pháp như: So sánh Pythagore, bất đẳng thức tam giác, nguyên lý trắc địa và so sánh đồ thị các hàm số. 2. Thiết lập được bất đẳng thức AM –GM từ ba số đến n số dương, bất đẳng thức Guha, bất đẳng thức Chebyshev, bất đẳng thức Schur và một số bất đẳng thức khác bằng cách sử dụng phương pháp nguyên lý bao hàm. 3. Vận dụng được các phương pháp nói trên vào việc giải các bài toán chứng minh một số bất đẳng thức cơ bản bằng phương pháp hình học và một số bài toán có liên quan.

pdf72 trang | Chia sẻ: builinh123 | Lượt xem: 2619 | Lượt tải: 5download
Bạn đang xem trước 20 trang tài liệu Luận văn Chứng minh một số bất đẳng thức cơ bản bằng phương pháp hình học, để xem tài liệu hoàn chỉnh bạn click vào nút DOWNLOAD ở trên
và hai hình chữ nhật trên trùng khít lên nhau khi hai cạnh 𝑎/𝑏 và 𝑐/𝑑 bằng nhau. Ví dụ 2.1.2. Bất đẳng thức AM-GM có thể được minh họa bằng cách sử dụng các tam giác vuông cân có diện tích là các số dương 𝑎 và 𝑏: 1 2 (√𝑎) 2 + 1 2 (√𝑏) 2 ≥ √𝑎.√𝑏⇔ 𝑎 + 𝑏 2 ≥ √𝑎𝑏 26 Hình 2.3 Phép tính sấp sỉ các căn bậc hai của Heron Trong cuốn sách Metrika của ông Heron (Nửa thứ hai của thế kỉ thứ nhất) đã đưa ra một phương pháp về phép tính sấp xỉ căn bậc hai √𝑛 của một số nguyên n: Nếu 𝑛 = 𝑎𝑏 thì √𝑛 sấp sỉ (𝑎+𝑏) 2 và phép tích xấp sỉ này chính xác nhất khi a gần bằng b vì √𝑛 = √𝑎𝑏 ≤ ( 𝑎+𝑏 2 ). Trong thực tế nếu 𝑥1 là một giá trị sấp sỉ của √𝑛 thì 𝑥2 = [𝑥1+( 𝑛 𝑥1 )] 2 là số gần với √𝑛 hơn 𝑥1Phương pháp của Newton dựa trên ý tưởng này. Ví dụ 2.1.3. Đối với bốn số dương bất kỳ 𝑎, 𝑏, 𝑐, 𝑑 với 𝑎 ≤ 𝑏, 𝑐 ≤ 𝑑, ta có 𝑎𝑑 + 𝑏𝑐 ≤ 𝑎𝑐 + 𝑏𝑑. Hình 2.4 Bất đẳng thức cũng đúng nếu 𝑎 ≥ 𝑏 và 𝑐 ≥ 𝑑. Đẳng thức xảy ra khi 𝑎 = 𝑏 hoặc 𝑐 = 𝑑. Nếu 𝑎 = 𝑐 = √𝑥 và 𝑏 = 𝑑 = √𝑦 thì chúng ta có thêm bằng chứng khác về bất đẳng thức AM- GM. Nếu đặt 𝑐 = 𝑎2 và 𝑑 = 𝑏2 thì ta có 𝑎2𝑏 + 𝑎𝑏2 ≤ 𝑎3 + 𝑏3. Bất đẳng thức này cho ta hình ảnh không gian ba chiều của môṭ hình lập phương. Trong hình 2.5 chúng ta nhận Thang Long University Library 27 thấy rằng 𝑎 ≤ 𝑏 dẫn đến 𝑎2𝑏 + 𝑎𝑏2 ≤ 𝑎3 + 𝑏3. Từ đó ta có thể mở rộng ý tưởng trong hình 2.4 bằng việc sử dụng thể tích của hình hộp. Hình 2.5 2.2 Bất đẳng thức Chebyshev Kết quả của ví dụ 2.1.3 được sử dụng để thiết lập bất đẳng thức Chebyshev (Pafnuty Lvovich Chebyshev, 1821-1894): Định lý 2.2. Cho 𝑛 ≥ 2 và 0 < 𝑥1 ≤ 𝑥2 ≤ ⋯ ≤ 𝑥𝑛, khi đó (i) Nếu 0 < 𝑦1 ≤ 𝑦2 ≤ ⋯ ≤ 𝑦𝑛, thì ∑ 𝑥𝑖 𝑛 𝑖=1 ∑ 𝑦𝑖 ≤ 𝑛∑ 𝑥𝑖𝑦𝑖 𝑛 𝑖=1 𝑛 𝑖=1 (2.2𝑎). (ii) Nếu 𝑦1 ≥ 𝑦2 ≥ ⋯ ≥ 𝑦𝑛 > 0, thì ∑ 𝑥𝑖 𝑛 𝑖=1 ∑ 𝑦𝑖 ≥ 𝑛∑ 𝑥𝑖𝑦𝑖 𝑛 𝑖=1 𝑛 𝑖=1 (2.2𝑏). Đẳng thức xảy ra khi và chỉ khi 𝑥1 = 𝑥2 = ⋯ = 𝑥𝑛 hoặc 𝑦1 = 𝑦2 = ⋯ = 𝑦𝑛 . Chứng minh. + TH (𝑖), sử dụng kết quả trong ví dụ 2.1.3 với 𝑎 = 𝑥𝑖 , 𝑏 = 𝑥𝑗 , 𝑐 = 𝑦𝑖 , 𝑑 = 𝑦𝑗 ta đươc̣ 𝑥𝑖𝑦𝑗 + 𝑥𝑗𝑦𝑖 ≤ 𝑥𝑖𝑦𝑖 + 𝑥𝑗𝑦𝑗 ..Khi đó như minh họa trong hình 2.6, mỗi cặp hình chữ nhật có phần che khuất có diêṇ tích nhỏ hơn hoăc̣ bằng diêṇ tích của môṭ căp̣ hình chữ nhâṭ có tổng diêṇ tích 𝑥𝑖𝑦𝑖 + 𝑥𝑗𝑦𝑗 và do đó (𝑥1 + 𝑥2 +⋯+ 𝑥𝑛)(𝑦1 + 𝑦2 +⋯+ 𝑦𝑛) ≤ 𝑛(𝑥1𝑦1 + 𝑥2𝑦2 +⋯+ 𝑥𝑛𝑦𝑛). 28 Hình 2.6 + TH (𝑖𝑖), chúng ta sử duṇg kết quả trong ví du ̣2.1.3 với 𝑎 = 𝑥𝑖 , 𝑏 = 𝑥𝑗 , 𝑐 = 𝑦𝑗 , 𝑑 = 𝑦𝑖 ta đươc̣ 𝑥𝑖𝑦𝑗 + 𝑥𝑗𝑦𝑖 ≥ 𝑥𝑖𝑦𝑖 + 𝑥𝑗𝑦𝑗 . Do đó ta đươc̣ bất đẳng thức ngươc̣ chiều, và (𝑥1 + 𝑥2 +⋯+ 𝑥𝑛). (𝑦1 + 𝑦2 +⋯+ 𝑦𝑛) ≥ 𝑛(𝑥1𝑦1 + 𝑥2𝑦2 +⋯𝑥𝑛𝑦𝑛). Đẳng thức xảy ra ở cả hai trường hợp trên khi và chỉ khi 𝑥1 = 𝑥2 = ⋯ = 𝑥𝑛 hoặc 𝑦1 = 𝑦2 = ⋯ = 𝑦𝑛 .(□). Đặc biệt trong TH (𝑖𝑖), khi thay 𝑦𝑖 = 1/𝑥𝑖 ta được (𝑥1 + 𝑥2 +⋯+ 𝑥𝑛) ( 1 𝑥1 + 1 𝑥2 +⋯+ 1 𝑥𝑛 ) ≥ 𝑛2. (2.2𝑐) Đẳng thức xẩy ra khi và chỉ khi 𝑥1 = 𝑥2 = ⋯ = 𝑥𝑛. Bài toán 2.2.1. Bất đẳng thức Nesbitt [11] Một chủ đề trong các cuôc̣ thi toán đa ̃đưa ra bài toán: Nếu 𝑎, 𝑏, 𝑐 là các số dương thì 𝑎 𝑏 + 𝑐 + 𝑏 𝑐 + 𝑎 + 𝑐 𝑎 + 𝑏 ≥ 3 2 Chứng minh. Áp dụng (2.2𝑐) cho ba số 𝑎 + 𝑏, 𝑏 + 𝑐, 𝑐 + 𝑎: Thang Long University Library 29 𝑎 𝑏 + 𝑐 + 𝑏 𝑐 + 𝑎 + 𝑐 𝑎 + 𝑏 = ( 𝑎 𝑏 + 𝑐 + 1) + ( 𝑏 𝑐 + 𝑎 + 1) + ( 𝑐 𝑎 + 𝑏 + 1) − 3 = (𝑎 + 𝑏 + 𝑐) ( 1 𝑎 + 𝑏 + 1 𝑏 + 𝑐 + 1 𝑐 + 𝑎 ) − 3 = 1 2 [(𝑎 + 𝑏) + (𝑏 + 𝑐) + (𝑐 + 𝑎)] ( 1 𝑎 + 𝑏 + 1 𝑏 + 𝑐 + 1 𝑐 + 𝑎 ) − 3 ≥ ( 1 2 . 9) − 3 = 3 2 . Đẳng thức xảy ra khi 𝑎 = 𝑏 = 𝑐. Vậy bất đẳng thức Nesbitt đã được chứng minh. Bài toán 2.2.2. Bất đẳng thức Voicu [15] Cho 𝛼, 𝛽, 𝛾 biểu thi ̣các góc đươc̣ taọ bởi đường chéo với các caṇh gồm chiều dài, rôṇg và chiều cao của hình hôp̣ chữ nhâṭ như minh hoạ trong hình 2.7. Chứng minh rằng tan𝛼 tan𝛽 tan 𝛾 ≥ 2√2. Đẳng thức xảy ra nếu và chỉ nếu hình hộp là một hình lập phương. Hiǹh 2.7 Chứng minh. Biến đổi vế trái, ta có 30 𝑉𝑇 = tan𝛼 tan𝛽 tan 𝛾 = √𝑏2 + 𝑐2 𝑎 . √𝑎2 + 𝑐2 𝑏 . √𝑎2 + 𝑏2 𝑐 ≥ √2𝑏𝑐 𝑎 . √2𝑎𝑐 𝑏 . √2𝑎𝑏 𝑐 = 2√2 𝑎𝑏𝑐 𝑎𝑏𝑐 = 2√2. Đẳng thức xảy ra khi 𝑎 = 𝑏 = 𝑐 suy ra tan𝛼 = tan𝛽 = tan 𝛾 =√2. Hay hình hộp chữ nhật đã cho là hình lập phương. Vậy bất đẳng thức Voicu đã được chứng minh. - Từ hình 2.7, ta cũng có 𝑐𝑜𝑠2𝛼 + 𝑐𝑜𝑠2𝛽 + 𝑐𝑜𝑠2𝛾 = 𝑎2 𝑎2 + 𝑏2 + 𝑐2 + 𝑏2 𝑎2 + 𝑏2 + 𝑐2 + 𝑐2 𝑎2 + 𝑏2 + 𝑐2 = 1. Cho 𝑥 = 𝑐𝑜𝑠𝛼, 𝑦 = 𝑐𝑜𝑠𝛽, 𝑧 = 𝑐𝑜𝑠𝛾, khi đó 𝑥2 + 𝑦2 + 𝑧2 = 1. Áp dụng bất đẳng thức (2.2𝑐) cho ba số 𝑥2, 𝑦2 và 𝑧2, ta có 1 + 𝑡𝑎𝑛2𝛼 𝑡𝑎𝑛2𝛽 𝑡𝑎𝑛2𝛾 = 1 + 1 − 𝑥2 𝑥2 . 1 − 𝑦2 𝑦2 . 1 − 𝑧2 𝑧2 = 𝑥2𝑦2 + 𝑦2𝑧2 + 𝑧2𝑥2 𝑥2𝑦2𝑧2 = 1 𝑥2 + 1 𝑦2 + 1 𝑧2 = (𝑥2 + 𝑦2 + 𝑧2) ( 1 𝑥2 + 1 𝑦2 + 1 𝑧2 ) ≥ 9. Đẳng thức xảy ra khi và chỉ khi { 𝑥 = 𝑦 = 𝑧 = 1 √3 𝑥2 + 𝑦2 + 𝑧2 = 1 Suy ra tan𝛼 = tan𝛽 = tan 𝛾 = √2. Nghiã là hình hôp̣ đa ̃ cho ở trên là hình lâp̣ phương. 2.3 Các bất đẳng thức AM-GM cho ba số Thiết lâp̣ bất đẳng thức AM- GM √𝑥𝑦𝑧 3 ≤ 𝑥+𝑦+𝑧 3 cho các số dương 𝑥, 𝑦, 𝑧. Khi đó đăṭ 𝑥 = 𝑎3, 𝑦 = 𝑏3, 𝑧 = 𝑐3, ta có bất đẳng thức trên tương đương với bất đẳng thức 3𝑎𝑏𝑐 ≤ 𝑎3 + 𝑏3 + 𝑐3. Thang Long University Library 31 Bổ đề 2.3.a. Cho các số 𝑎, 𝑏, 𝑐 ≥ 0, ta luôn có 𝑎𝑏 + 𝑏𝑐 + 𝑎𝑐 ≤ 𝑎2 + 𝑏2 + 𝑐2. Hiǹh 2.8 Chứng minh. Không mất tính tổng quát, ta có thể giả sử 𝑎 ≥ 𝑏 ≥ 𝑐 và từ hình 2.8 ta thấy diện tích 𝑎𝑏 + 𝑏𝑐 + 𝑎𝑐 của ba hình chữ nhật bên trái nhỏ hơn hoặc bằng diện tích 𝑎2 + 𝑏2 + 𝑐2 của ba hình vuông bên phải và chúng trùng khít lên nhau khi ba kích thước 𝑎, 𝑏, 𝑐 đôi một bằng nhau. (□). Ngoài ra bất đẳng thức ở Bổ đề 2.3.a cũng có thể được chứng minh bằng việc áp dụng bất đẳng thức AM- GM cho các cặp số: 𝑎2 và 𝑏2; 𝑏2 và 𝑐2; 𝑐2 và 𝑎2. Bài toán 2.3.1. Bất đẳng thức của Guba [6] Cho a, b, c biểu thi ̣chiều dài, chiều rôṇg, chiều cao của môṭ hình hôp̣ chữ nhâṭ khi đó goị 𝐾1 = 𝑎𝑏,𝐾2 = 𝑏𝑐, 𝐾3 = 𝑎𝑐 là diêṇ tích các măṭ bên, 𝑉 = 𝑎𝑏𝑐 là thể tích và 𝑑 = √𝑎2 + 𝑏2 + 𝑐2 là đường chéo của khối hôp̣. Khi đó bất đẳng thức Guba chỉ ra rằng 𝐾1 2 + 𝐾2 2 + 𝐾3 2 ≥ √3𝑉𝑑. Chứng minh. Sử duṇg bổ đề 2.3.𝑎 ta có (𝑎 + 𝑏 + 𝑐)2 = 𝑎2 + 𝑏2 + 𝑐2 + 2(𝑎𝑏 + 𝑏𝑐 + 𝑎𝑐) ≥ 3(𝑎𝑏 + 𝑏𝑐 + 𝑎𝑐). 32 Và do đó ((𝐾1 2 + 𝐾2 2 + 𝐾3 2)2 ≥ 3(𝐾1 2𝐾2 2 + 𝐾2 2𝐾3 2 + 𝐾3 2𝐾1 2) = 3𝑎2𝑏2𝑐2(𝑎2 + 𝑏2 + 𝑐2) = 3𝑉2𝑑2. Hay 𝐾1 2 + 𝐾2 2 + 𝐾3 2 ≥ √3𝑉𝑑. Đẳng thức xảy ra khi 𝑎 = 𝑏 = 𝑐. Vâỵ bất đẳng thức Guba đa ̃đươc̣ chứng minh. Định lý 2.3.b. Cho 𝑎, 𝑏, 𝑐 ≥ 0, 3𝑎𝑏𝑐 ≤ 𝑎3 + 𝑏3 + 𝑐3. Hình 2.9. Chứng minh. [2]. Trong hình 2.9 riêng các hình chữ nhâṭ màu trắng ở bên trái có diện tích khác với diện tích các hình chữ nhật màu trắng ở bên phải. Còn các hình chữ nhật còn lại ở cả hai hình được tô màu giống nhau và kẻ đường chéo giống nhau thì có diêṇ tích giống nhau, theo bổ đề 2.3.𝑎 ta có chiều cao của hình chữ nhâṭ bên trái nhỏ hơn hoăc̣ bằng chiều cao của hình chữ nhâṭ bên phải và ba hình chữ nhâṭ màu trắng ở bên trái có diêṇ tích là 3𝑎𝑏𝑐 nhỏ hơn hoăc̣ bằng diêṇ tích 𝑎3 + 𝑏3 + 𝑐3của ba hình chữ nhâṭ màu trắng ở bên phải và hai hình chữ nhật trùng khít lên nhau khi và chỉ khi ba kích thước 𝑎, 𝑏, 𝑐 đôi một bằng nhau. Vậy định lý 2.3. 𝑏 đã được chứng minh. Từ đó ta có bất đẳng thức AM-GM cho ba số không âm 𝑎, 𝑏, 𝑐 tương đương với bất đẳng thức trong định lí 2.3.𝑏. Ngoài ra ta Thang Long University Library 33 cũng có thể minh họa bất đẳng thức này bằng cách dùng thể tích của ba hình hôp̣ chữ nhâṭ cùng có ba kích thước là 𝑎, 𝑏, 𝑐 so sánh với thể tích của ba hình lâp̣ phương có cạnh tương ứng là 𝑎, 𝑏, 𝑐 hoặc ta minh họa bằng hình ảnh không gian ba chiều trong hình 2.10 và ta thấy rằng thể tích 𝑎𝑏𝑐 của hình hôp̣ chữ nhật với đô ̣ dài các caṇh là 𝑎 ≥ 𝑏 ≥ 𝑐 nhỏ hơn hoặc bằng thể tích của ba hình chóp có đáy là hình vuông cạnh lần lượt là 𝑎, 𝑏, 𝑐 và chiều cao của ba chóp theo thứ tư ̣đó cũng là 𝑎, 𝑏, 𝑐. Ba hình chóp nói trên được lắp vào một hình lập phương cạnh bằng a (minh hoạ hình 2.10). Do đó 𝑎𝑏𝑐 ≤ 1 3 𝑎2. 𝑎 + 1 3 𝑏2. 𝑏 + 1 3 𝑐2. 𝑐. Hình 2.10 Bài toán 2.3.2. Trong tất cả các hình tam giác có cùng chu vi, tam giác đều có diện tích lớn nhất. Chứng minh. Xét môṭ hình tam giác có đô ̣dài các caṇh là 𝑎, 𝑏, 𝑐, diêṇ tích là 𝐾 và nửa chu vi là 𝑠 = 𝑎+𝑏+𝑐 2 . Áp duṇg bất đẳng thức AM- GM cho ba số 𝑠 − 𝑎, 𝑠 − 𝑏, 𝑠 − 𝑐, ta có √(𝑠 − 𝑎)(𝑠 − 𝑏)(𝑠 − 𝑐) 3 ≤ (𝑠 − 𝑎) + (𝑠 − 𝑏) + (𝑠 − 𝑐) 3 = 𝑠 3 . 34 Hay (𝑠 − 𝑎)(𝑠 − 𝑏)(𝑠 − 𝑐) ≤ 𝑠3/27. Đẳng thức xảy ra khi và chỉ khi 𝑎 = 𝑏 = 𝑐. Áp dụng công thức tính diện tích của Heron 𝐾 = √𝑠(𝑠 − 𝑎)(𝑠 − 𝑏)(𝑠 − 𝑐). ⇔𝐾2 = 𝑠(𝑠 − 𝑎)(𝑠 − 𝑏)(𝑠 − 𝑐) = 𝑠 [√(𝑠 − 𝑎)(𝑠 − 𝑏)(𝑠 − 𝑐) 3 ] 3 ≤ 𝑆4 27 . ⇔ 𝐾 ≤ √3𝑆2 9 . Vì chu vi là cố định nên s là cố định, do đó tam giác có diện tích lớn nhất là tam giác đều. (□). Bài toán 2.3.3. Tìm chiều cao và thể tích của hình trụ nội tiếp được trong một hình nón có bán kính đáy 𝑅 và chiều cao 𝐻. Lời giải. Goị 𝑟 và ℎ biểu thi ̣ tương ứng bán kính đáy và chiều cao của hình trụ. Nếu ta gắn hình nón vào măṭ phẳng toạ đô ̣sao cho tâm của măṭ đáy trùng với gốc toạ đô,̣ chiều cao của nón nằm trên truc̣ (𝑜𝑦) và bán kính của nón nằm trên truc̣ (𝑜𝑥), khi đó điểm (𝑟, ℎ) nằm trên đường thẳng 𝑥 𝑅 + 𝑦 𝐻 = 1 do đó 𝑟 𝑅 + ℎ 𝐻 = 1, xem hình 2.11. Hình 2.11 Từ đó thể tích của hình tru ̣là 𝑉 = 𝜋 𝑟2ℎ và ta có Thang Long University Library 35 𝑉 = 𝜋𝑟2ℎ = 4𝜋𝑅2𝐻. 𝑟 2𝑅 . 𝑟 2𝑅 . ℎ 𝐻 ≤ 4𝜋𝑅2𝐻.( 𝑟 2𝑅 + 𝑟 2𝑅 + ℎ 𝐻 3 ) 3 = 4𝜋𝑅2𝐻. 1 27 = 4 9 . 1 3 𝜋𝑅2𝐻. Đẳng thức xảy ra khi và chỉ khi 𝑟 2𝑅 = ℎ 𝐻 . Từ đẳng thức 𝑟 𝑅 + ℎ 𝐻 = 1, ta thấy hình tru ̣có thể tích lớn nhất khi 𝑟 = 2𝑅 3 , ℎ = 𝐻 3 . Và thể tích của hình tru ̣ bằng 4/9 thể tích của hình nón. Bài toán 2.3.4. Cực trị của tổng và tích hàm số tang Xét môṭ tam giác với các góc đo 𝛼, 𝛽, 𝛾 ta thấy rằng tan 𝛼 2 tan 𝛽 2 tan 𝛾 2 ≤ √3 9 và tan 𝛼 2 + tan 𝛽 2 + tan 𝛾 2 ≥ √3. Vì 𝛼 + 𝛽 + 𝛾 = 𝜋, tan 𝛼+𝛽 2 = cot 𝛾 2 nên tan 𝛼 2 +tan 𝛽 2 1−tan 𝛼 2 tan 𝛽 2 = 1 tan 𝛾 2 . Và do đó: tan 𝛼 2 tan 𝛽 2 + tan 𝛽 2 tan 𝛾 2 + tan 𝛾 2 tan 𝛼 2 = 1. Bây giờ chúng ta đăṭ 𝑥 = tan 𝛼 2 , 𝑦 = tan 𝛽 2 , 𝑧 = tan 𝛾 2 . Bài toán trên tương đương với bài toán chứng minh 𝑥𝑦𝑧 ≤ √3 9 và 𝑥 + 𝑦 + 𝑧 ≥ √3 khi 𝑥𝑦 + 𝑦𝑧 + 𝑧𝑥 = 1. Chứng minh Áp duṇg bất đẳng thức AM- GM cho ba số 𝑥𝑦, 𝑦𝑧 và 𝑥𝑧, ta có 1 3 = 𝑥𝑦 + 𝑦𝑧 + 𝑧𝑥 3 ≥ √𝑥𝑦. 𝑦𝑧. 𝑧𝑥 3 = (𝑥𝑦𝑧)2/3. Từ đó ta suy ra 𝑥𝑦𝑧 ≤ √3 9 . Theo kết quả của bổ đề 2.3.𝑎, ta có (𝑥 + 𝑦 + 𝑧)2 = 𝑥2 + 𝑦2 + 𝑧2 + 2(𝑥𝑦 + 𝑦𝑧 + 𝑧𝑥) ≥ 3(𝑥𝑦 + 𝑦𝑧 + 𝑧𝑥)(2.3). 36 và từ 𝑥𝑦 + 𝑦𝑧 + 𝑧𝑥 = 1, ta có 𝑥 + 𝑦 + 𝑧 ≥ √3. Đẳng thức xảy ra trong cả hai bất đẳng thức trên khi và chỉ khi 𝑥 = 𝑦 = 𝑧 = √3 3 , tức là khi tam giác đa ̃cho là tam giác đều. (□). Bài toán 2.3.5. Bất đẳng thức của Newton Cho n số thưc̣ 𝑎1, 𝑎2, , 𝑎𝑛 và môṭ chỉ số i cố điṇh, 0 ≤ 𝑖 ≤ 𝑛, 𝑖 thuôc̣ hàm đối xứng 𝜎𝑖 , 𝜎𝑖 đươc̣ điṇh nghiã là hê ̣số của 𝑥 𝑖−1 trong khai triển (𝑥 + 𝑎1)(𝑥 + 𝑎2) (𝑥 + 𝑎𝑛). Kết hơp̣ với mỗi 𝜎𝑖 là 𝑖 thuôc̣ giá tri ̣ trung bình đối xứng 𝑆𝑖, đươc̣ điṇh nghiã là 𝑆𝑖 = 𝜎𝑖/( 𝑛 𝑖 ). Ví du ̣khi 𝑛 = 3, ta có 𝑆0 = 1, 𝑆1 = (𝑎1+𝑎2+𝑎3) 3 , 𝑆2 = (𝑎1𝑎2+𝑎2𝑎3+𝑎3𝑎1) 3 và 𝑆3 = 𝑎1𝑎2𝑎3. Bất đẳng thức Newton (Isaac Newton, 1642-1727) đã được thiết lập là 𝑆𝑖−1𝑆𝑖 ≤ 𝑆𝑖 2. Bây giờ chúng ta se ̃đưa ra môṭ chứng minh ngắn cho trường hơp̣ đăc̣ biêṭ với 𝑛 = 3, 𝑖 = 2 và 𝑎1, 𝑎2, 𝑎3 ≥ 0, ta có 1 3 (𝑎1 + 𝑎2 + 𝑎3)𝑎1𝑎2𝑎3 ≤ ((𝑎1𝑎2 + 𝑎2𝑎3 + 𝑎3𝑎1)/3) 2 . Nếu chúng ta đăṭ 𝑥 = 𝑎1𝑎2, 𝑦 = 𝑎2𝑎3, 𝑧 = 𝑎3𝑎1thì bất đẳng thức trên tương đương với bất đẳng thức 3(𝑥𝑦 + 𝑦𝑧 + 𝑥𝑧) ≤ (𝑥 + 𝑦 + 𝑧)2, đó cũng chính là kết quả của (2.3). Bài toán bất đẳng thức trong ngành hàng không Để tránh việc xếp hàng dài, tăng lệ phí và lãng phí thời gian kiểm tra hành lí tại sân bay, nhiều hành khách đã lựa chọn cách chỉ mang theo va li hành lí. Tuy nhiên các hãng hàng không đều có giới hạn về kích cỡ và trọng lượng cho hành lí mang theo. Đặc biệt là giới hạn về kích thước 𝑥, 𝑦, 𝑧 của túi hành lí có (𝑥 + 𝑦 + 𝑧) không vượt quá 45 inches. Bất đẳng thức AM- GM chỉ ra rằng hình dạng của những va li với thể tích lớn nhất là một khối lập phương có cạnh là 15 inches. Thế nhưng một túi hành lí như vậy hiếm khi vừa một cái ngăn chứa đồ ở trên đầu. Thang Long University Library 37 Chúng ta có thể mở rôṇg bất đẳng thức AM-GM cho bốn số dương 𝑎, 𝑏, 𝑐, 𝑑, ta có 𝑎 + 𝑏 + 𝑐 + 𝑑 4 = 1 2 ( 𝑎 + 𝑏 2 + 𝑐 + 𝑑 2 ) ≥ 1 2 (√𝑎𝑏 + √𝑐𝑑) ≥ √√𝑎𝑏√𝑐𝑑 ≥ √𝑎𝑏𝑐𝑑 4 . Tương tư ̣ta có thể mở rôṇg bất đẳng thức này cho 𝑛 số khi 𝑛 là lũy thừa của 2 (xem bài tập 3.2.2 - chương III). Để mở rôṇg bất đẳng thức cho tất cả số nguyên dương 𝑛, trước hết chúng ta chứng minh bất đẳng thức dưới đây. 2.4 Bất đẳng thức của Guha Bất đẳng thức của Guha chỉ ra rằng nếu 𝑎 ≥ 0, 𝑝 ≥ 𝑞 > 0 và 𝑥 ≥ 𝑦 > 0 thì (𝑝𝑥 + 𝑦 + 𝑎)(𝑥 + 𝑞𝑦 + 𝑎) ≥ [(𝑝 + 1)𝑥 + 𝑎][(𝑞 + 1)𝑦 + 𝑎]. Hình 2.12. Chứng minh Từ hình 2.12 ta thấy các hình chữ nhật được tô màu giống nhau và có các đường kẻ chéo giống nhau thì có diện tích bằng nhau. Từ đó hình chữ nhật bên trái thì lắp vừa bên trong hình chữ nhật bên phải. Do đó diện tích [(𝑝 + 1)𝑥 + 𝑎][(𝑞 + 1)𝑦 + 𝑎] hình chữ nhật bên phải lớn hơn hoặc bằng diện tích (𝑝𝑥 + 𝑦 + 𝑎)(𝑥 + 𝑞𝑦 + 𝑎) của hình chữ nhật bên 38 trái. Hai hình chữ nhật trên trùng khít lên nhau khi và chỉ khi hai kích thước 𝑥, 𝑦 bằng nhau. (□) Ngoài ra theo phương pháp đaị số, bất đẳng thức trên tương đương với bất đẳng thức (𝑝𝑥 − 𝑞𝑦)(𝑥 − 𝑦) ≥ 0, điều đó có nghiã là diêṇ tích của hình chữ nhâṭ nhỏ màu trắng ở bên phải hình 2.12 là không âm. Do đó, đẳng thức xảy ra nếu và chỉ nếu 𝑥 = 𝑦. 2.5 Bất đẳng thức AM-GM cho n số Bất đẳng thức AM- GM cho n số dương 𝑥1, 𝑥2, , 𝑥𝑛 phát biểu rằng 𝐺𝑛 ≤ 𝐴𝑛, trong đó 𝐴𝑛 = (𝑥1+𝑥2+⋯+𝑥𝑛) 𝑛 và 𝐺𝑛 = (𝑥1. 𝑥2𝑥𝑛) 1/𝑛. Không mất tính tổng quát ta giả sử 𝑥1 ≤ 𝑥2 ≤ ⋯ ≤ 𝑥𝑛. Bất đẳng thức Guha được áp dụng để thiết lập bất đẳng thức AM-GM cho 𝑛 số. Chúng ta minh hoạ cho 𝑛 = 4. Quá trình này cũng giống hệt đối với mọi 𝑛 ≥ 2 bất kì. Ta có (4𝐴4) 4 = (𝑎 + 𝑏 + 𝑐 + 𝑑)(𝑎 + 𝑏 + 𝑐 + 𝑑)(𝑎 + 𝑏 + 𝑐 + 𝑑)(𝑎 + 𝑏 + 𝑐 + 𝑑) ≥ (2𝑎 + 𝑐 + 𝑑)(2𝑏 + 𝑐 + 𝑑)(𝑎 + 𝑏 + 𝑐 + 𝑑)(𝑎 + 𝑏 + 𝑐 + 𝑑) ≥ (3𝑎 + 𝑑)(2𝑏 + 𝑐 + 𝑑)(𝑏 + 2𝑐 + 𝑑)(𝑎 + 𝑏 + 𝑐 + 𝑑) ≥ (4𝑎)(2𝑏 + 𝑐 + 𝑑)(𝑏 + 2𝑐 + 𝑑)(𝑏 + 𝑐 + 2𝑑) ≥ (4𝑎)(3𝑏 + 𝑑)(3𝑐 + 𝑑)(𝑏 + 𝑐 + 2𝑑) ≥ (4𝑎)(4𝑏)(3𝑐 + 𝑑)(𝑐 + 3𝑑). ≥ (4𝑎)(4𝑏)(4𝑐)(4𝑑) = (4𝐺4) 4 Đẳng thức xảy ra khi và chỉ khi 𝑎 = 𝑏 = 𝑐 = 𝑑. (□). Ta cũng áp duṇg liên tiếp bất đẳng thức Guha để chứng minh bất đẳng thức AM – GM cho 𝑛 số. Thang Long University Library 39 Định lý 2.5.1. Trong tất cả các hình tam giác nôị tiếp đươc̣ trong đường tròn bán kính 𝑅 thì tam giác đều có diêṇ tích lớn nhất. Chứng minh. Trước tiên ta cần chú ý rằng trong tất cả các hình tam giác nôị tiếp đươc̣ trong đường tròn bán kính 𝑅 sẽ có môṭ tam giác cân có chiều cao cao hơn và do đó nó se ̃có diêṇ tích lớn hơn, như minh hoạ trong hình 2.13𝑎. Vì vâỵ chúng ta chỉ cần xét tam giác cân. Hình 2.13 Goị 𝑅 là bán kính của đường tròn ngoaị tiếp tam giác cân có đô ̣dài caṇh đáy là 𝑏 và chiều cao là ℎ, đươc̣ minh hoạ trong hình 2.13𝑏. Ta có thể giả sử ℎ ≥ 𝑅. Khi đó (𝑏/2)2 + (ℎ − 𝑅)2 = 𝑅2 hay (𝑏/2)2 = ℎ(2𝑅 − ℎ). Nếu 𝐾 biểu thi ̣diêṇ tích của hình tam giác thì 𝐾 = 1 2 𝑏ℎ ⇔𝐾2 = ( 𝑏ℎ 2 ) 2 = ℎ3(2𝑅 − ℎ) = 27( ℎ 3 ) 3 (2𝑅 − ℎ) ≤ 27 [ 1 4 ( ℎ 3 + ℎ 3 + ℎ 3 + (2𝑅 − ℎ))] 4 = 27 ( 𝑅 2 ) 4 ⇔𝐾 ≤ 3√3 4 𝑅2. 𝐾 đaṭ giá tri ̣lớn nhất bằng 3√3 4 𝑅2 khi và chỉ khi ℎ 3 = 2𝑅 − ℎ⇔ℎ = 3 2 𝑅. Hay tam giác đều có diện tích lớn nhất.(□). 40 Hệ quả 2.5.2. Nếu môṭ tam giác có diêṇ tích bằng 𝐾 nôị tiếp trong đường tròn có bán kính 𝑅 thì K ≤ 3√3 4 𝑅2. 2.6 Bất đẳng thức HM-AM-GM-RMS cho n số Chúng ta có thể dê ̃ dàng mở rôṇg bất đẳng thức AM – GM cho 𝐻𝑛 ≤ 𝐺𝑛 ≤ 𝐴𝑛 ≤ 𝑅𝑛 trong đó 𝐻𝑛 biểu thi ̣ cho trung bình điều hòa (HM) của 𝑥1, 𝑥2, , 𝑥𝑛, 𝐻𝑛 = 𝑛 1 𝑥1 + 1 𝑥2 +⋯+ 1 𝑥𝑛 . Và Rn biểu thi ̣ căn bâc̣ hai của trung bình bình phương (RMS) của 𝑥1, 𝑥2, , 𝑥𝑛, 𝑅𝑛 = √ 𝑥1 2 + 𝑥2 2 +⋯+ 𝑥𝑛2 𝑛 . Chứng minh 𝐻𝑛 ≤ 𝐺𝑛 ≤ 𝐴𝑛 ≤ 𝑅𝑛 - Áp duṇg bất đẳng thức AM – GM cho 1 𝑥1 , 1 𝑥2 , , 1 𝑥𝑛 số dương Ta có 1 𝑛 ( 1 𝑥1 + 1 𝑥2 +⋯+ 1 𝑥𝑛 ) ≥ √ 1 𝑥1 . 1 𝑥2 1 𝑥𝑛 𝑛 . Hay 𝐻𝑛 ≤ 𝐺𝑛 (1) - Không mất tính tổng quát ta có thể giả sử 𝑥1 ≤ 𝑥2 ≤ ⋯ ≤ 𝑥𝑛. Đăṭ 𝑦𝑖 = 𝑥𝑖 và áp duṇg (2.2𝑎) ta đươc̣ (𝑥1 + 𝑥2 +⋯+ 𝑥𝑛) 2 ≤ 𝑛(𝑥1 2 + 𝑥2 2 +⋯+ 𝑥𝑛 2). Hay 𝐴𝑛 ≤ 𝑅𝑛(2). - 𝐺𝑛 ≤ 𝐴𝑛 ( xem ở mục 2.5) (3). Từ(1), (2), (3), ta có 𝐻𝑛 ≤ 𝐺𝑛 ≤ 𝐴𝑛 ≤ 𝑅𝑛. (□). Từ đó ta có phát biểu của định lý 2.6 dưới đây. Thang Long University Library 41 Điṇh lí 2.6. Cho n số dương 𝑥1, 𝑥2, , 𝑥𝑛, ta có 𝑛 1 𝑥1 + 1 𝑥2 +⋯+ 1 𝑥𝑛 ≤ √𝑥1𝑥2𝑥𝑛 𝑛 ≤ 𝑥1 + 𝑥2 +⋯+ 𝑥𝑛 𝑛 ≤ √ 𝑥1 2 + 𝑥2 2 +⋯+ 𝑥𝑛2 𝑛 . 2.7 Tính chất của số Mediant và nghịch lý của Simpson Mediant của hai phân số 𝑎/𝑏 và 𝑐/𝑑, trong đó 𝑏 > 0 và 𝑑 > 0 là phân số 𝑎+𝑐 𝑏+𝑑 . Nếu 𝑎 𝑏 < 𝑐 𝑑 thì 𝑎 𝑏 < 𝑎+𝑐 𝑏+𝑑 < 𝑐 𝑑 . Bất đẳng thức này đươc̣ biết đến như là tính chất của số Mediant, chúng ta minh hoạ bất đẳng thức này (với 𝑎 > 0 và 𝑐 > 0) bằng bất đẳng thức tương đương 𝑎 𝑏 < 𝑎 𝑏+𝑑 + 𝑐 𝑏+𝑑 < 𝑐 𝑑 (∗) trong hình 2.14. (a) (b) (c) Hiǹh 2.14 Chứng minh bất đẳng thức (∗)  Từ hình 2.14 ta thấy - Ở hình (𝑏) hai hình chữ nhật có diện tích là 𝑎 𝑏+𝑑 + 𝑐 𝑏+𝑑 lớn hơn diện tích 𝑎 𝑏 của hình chữ nhật ở hình (𝑎). (1) 42 - Diện tích 𝑐 𝑑 của hình chữ nhật ở hình (𝑐) thì lớn hơn diện tích 𝑎 𝑏+𝑑 + 𝑐 𝑏+𝑑 của hình chữ nhật ở hình (𝑏). (2) Từ (1) và (2) ta có 𝑎 𝑏 < 𝑎 𝑏+𝑑 + 𝑐 𝑏+𝑑 < 𝑐 𝑑 . Vậy nếu 𝑎/𝑏 < 𝑐/𝑑 thì 𝑎 𝑏 < 𝑎+𝑐 𝑏+𝑑 < 𝑐 𝑑 . (□).  Ngoài ra môṭ phân số cũng có thể đươc̣ đaị diêṇ cho đường chéo của hình bình hành, điều này dâñ đến minh hoạ thứ hai thông qua quy tắc hình bình hành với phép côṇg véc tơ. Ở đây chúng ta thấy rằng đường chéo 𝑎+𝑐 𝑏+𝑑 thì nằm giữa hai caṇh kề nhau 𝑎 𝑏 , 𝑐 𝑑 và của hình bình hành cùng xuất phát từ gốc toạ đô.̣ Hiǹh 2.15 Xét thấy trung bình cộng là đơn điệu, nghiã là nếu 𝑥 < 𝑋 và 𝑦 < 𝑌 thì 𝑥+𝑦 2 < 𝑋+𝑌 2 nhưng mediant không đơn điệu điều này dâñ đến kết quả đáng ngac̣ nhiên được minh hoạ trong Bài toán 2.7 dưới đây. Chuquet, Poincaré và tính chất của số mediant Nicolas Chuquet, (khoảng 1450-1500) đã thảo luận mediant trong cuốn sách của ông Le Triparty en la science des Nombres, được xuất bản năm 1484. Chuquet đề cập đến phép tính này như qui tắc về các số trung bình. Vài thế kỉ sau đó nhà toán học nổi tiếng Henri Poincaré (1854- 1912) nói [3] “Nếu bạn muốn dạy phân số một cách trực quan thì bạn có thể minh Thang Long University Library 43 họa bằng cách chia cái bánh thành các phần bằng nhau hoặc bạn mang một quả táo đến lớp. Trong tất cả các trường hợp khác học sinh sẽ tiếp tục cộng thêm vào tử số và mẫu số.” Bài toán 2.7. Nghịch lý của Simpson Nghic̣h lý của Simpson đa ̃ đề câp̣ đến “Sư ̣ kết hơp̣ hoăc̣ so sánh giữa môṭ vài nhóm có thể đảo chiều khi các số liêụ đươc̣ kết hơp̣ để hình thành môṭ nhóm riêng”. Sư ̣so sánh thường đươc̣ thể hiêṇ bằng các phân số, tức là: 𝑎 𝑏 < 𝐴 𝐵 , 𝑐 𝑑 < 𝐶 𝐷 nhưng 𝑎+𝑐 𝑏+𝑑 > 𝐴+𝐶 𝐵+𝐷 . Bất đẳng thức trên còn đươc̣ goị là sư ̣ đảo chiều bất đẳng thức của Simpson. Ví du.̣ Giả sử môṭ trường đaị hoc̣ đang tuyển nhân viên cho hai khoa lic̣h sử và điạ lý. Có năm người đàn ông nôp̣ đơn vào khoa lic̣h sử và chỉ môṭ người đươc̣ tuyển, tám người phu ̣nữ nôp̣ đơn và hai người đươc̣ tuyển, tỉ lê ̣ thành công cho nam giới là 20% và tỉ lê ̣ thành công cho nữ giới là 25%, khoa sử đa ̃ ủng hô ̣ phu ̣ nữ hơn nam giới. Trong khoa điạ lý tám người đàn ông nôp̣ đơn và sáu người đươc̣ tuyển, năm người phu ̣nữ nôp̣ đơn và bốn người đươc̣ tuyển, tỉ lê ̣thành công của nam giới là 75% còn tỉ lê ̣thành công của nữ giới là 80%, khoa điạ lí đa ̃ủng hô ̣nữ giới hơn nam giới. Thế nhưng cả trường đaị hoc̣ có tổng là 13 nam và 13 nữ cùng nôp̣ đơn cho công viêc̣ thì 7 nam và 6 nữ đươc̣ tuyển, tỉ lê ̣thành công cho nam giới laị lớn hơn tỉ lê ̣thành công của nữ giới. Từ đó ta có 𝑎+𝑐 𝑏+𝑑 và 𝐴+𝐶 𝐵+𝐷 đều là các mediant của căp̣ 𝑎 𝑏 , 𝑐 𝑑 và căp̣ 𝐴 𝐵 , 𝐶 𝐷 . 44 Hình 2.16. Tuy nhiên nhìn vào hình 2.16 ta thấy 𝑎 𝑏 < 𝐴 𝐵 và 𝑐 𝑑 < 𝐶 𝐷 nhưng 𝑎+𝑐 𝑏+𝑑 > 𝐴+𝐶 𝐵+𝐷 . Từ đó ta có kết luận mediant là không đơn điệu. Ngoài ra viêc̣ sử duṇg quy tắc hình bình hành cũng đã giúp giải thích sư ̣đổi chiều của bất đẳng thức. 2.8 Trở laị với bất đẳng thức Chebyshev Bất đẳng thức Chebyshev (2.2𝑎) ở muc̣ 2.2 đa ̃có môṭ số ứng dụng rất hay mà chúng ta se ̃kiểm chứng ngay bây giờ, trước tiên chúng ta đưa ra môṭ chứng minh khác của (2.2𝑎) sử dụng diêṇ tích bao hàm trong hình 2.17. Hiǹh 2.17 Thang Long University Library 45 Chứng minh. Trong hình 2.17 mô phỏng 𝑛 hình chữ nhâṭ với các diêṇ tích 𝑥1𝑦1, 𝑥2𝑦2, , 𝑥𝑛𝑦𝑛 đươc̣ sắp xếp phủ chồng chéo lên môṭ hình chữ nhâṭ có các kích thước 𝑥1 + 𝑥2 +⋯+ 𝑥𝑛 và 𝑦1 + 𝑦2 +⋯+ 𝑦𝑛 vì vâỵ 𝑛(𝑥1𝑦1 + 𝑥2𝑦2 +⋯+ 𝑥𝑛𝑦𝑛) ≥ (𝑥1 + 𝑥2 +⋯+ 𝑥𝑛)(𝑦1 + 𝑦2 +⋯+ 𝑦𝑛) (2.8). (□) Để dễ hình dung ta chứng minh bất đẳng thức (2.8) của Chebyshev trong trường hợp 𝑛 = 4. Từ hình vẽ ta thấy các hình chữ nhật có diện tích lần lượt là 4𝑥1𝑦1, 4𝑥2𝑦2, 4𝑥3𝑦3, 4𝑥4𝑦4 được xắp xếp phủ chồng chéo lên hình chữ nhật có diện tích (𝑥1 + 𝑥2 + 𝑥3 + 𝑥4)(𝑦1 + 𝑦2 + 𝑦3 + 𝑦4). Vì vậy ta luôn có 4(𝑥1𝑦1 + 𝑥2𝑦2 + 𝑥3𝑦3 + 𝑥4𝑦4) ≥ (𝑥1 + 𝑥2 + 𝑥3 + 𝑥4)(𝑦1 + 𝑦2 + 𝑦3 + 𝑦4). (□). Bài toán 2.8.1. Trung bình của tổng các bình phương lớn hơn hoặc bằng bình phương của trung bình cộng. Cho tâp̣ hơp̣ các số thưc̣ {𝑎1, 𝑎2, , 𝑎𝑛} và đăṭ 𝑥𝑖 = 𝑦𝑖 = |𝑎𝑖|, theo bất đẳng thức Chebyshev ( 2.8), ta có 𝑛(𝑎1 2 + 𝑎2 2 +⋯+ 𝑎3 2) ≥ (|𝑎1| + |𝑎2| + ⋯+ |𝑎𝑛|) 2 ≥ (𝑎1 + 𝑎2 +⋯+ 𝑎𝑛) 2 Hay 46 1 𝑛 (𝑎1 2 + 𝑎2 2 +⋯+ 𝑎𝑛 2) ≥ ( 𝑎1 + 𝑎2 +⋯+ 𝑎𝑛 𝑛 ) 2 . Bất đẳng thức này măc̣ dù giống hêṭ bất đẳng thức giữa trung bình côṇg – căn bâc̣ hai của trung bình bình phương cho 𝑛 số dương tuy nhiên bất đẳng thức này còn maṇh vì nó áp duṇg đươc̣ cho tất cả các số thực 𝑎1, 𝑎2, , 𝑎𝑛. Bài toán 2.8.2. Các trung bình lũy thừa Nếu ta đổi biến công thức (2.8) hai lần, đầu tiên với 𝑦𝑖 = 𝑥𝑖 2 và sau đó với 𝑦𝑖 = 𝑥𝑖, ta có 𝑛2(𝑥1 3 + 𝑥2 3 +⋯+ 𝑥𝑛 3) ≥ (𝑥1 + 𝑥2 +⋯+ 𝑥𝑛). 𝑛(𝑥1 2 + 𝑥2 2 +⋯+ 𝑥𝑛 2) ≥ (𝑥1 + 𝑥2 +⋯+ 𝑥𝑛) 3 Hay 𝑥1 + 𝑥2 +⋯+ 𝑥𝑛 𝑛 ≤ √ 𝑥1 3 + 𝑥2 3 +⋯+ 𝑥𝑛 3 𝑛 3 . Biểu thức bên phải của bất đẳng thức trên đươc̣ biết đến như là trung bình lũy thừa, và phương pháp đó có thể làm lăp̣ đi lăp̣ laị để thiết lâp̣ bất đẳng thức giữa trung bình côṇg và trung bình lũy thừa bâc̣ 𝑝 cho các số dương 𝑥𝑖. Từ đó với bất cứ số nguyên 𝑝 ≥ 2, ta luôn có 𝑥1 + 𝑥2 +⋯+ 𝑥𝑛 𝑛 ≤ √ 𝑥1 𝑝 + 𝑥2 𝑝 +⋯+ 𝑥𝑛 𝑝 𝑛 𝑝 . Ngoài ra việc sử duṇg bất đẳng thức Bernoulli cũng có thể thiết lâp̣ bất đẳng thức trung bình lũy thừa (xem chứng minh định lý 2.8 dưới đây). Điṇh lý 2.8. Cho 𝑥1, 𝑥2, , 𝑥𝑛, 𝑝, 𝑞 là các số thưc̣ dương, với 𝑝 ≤ 𝑞, ta luôn có Thang Long University Library 47 √ 𝑥1 𝑝 + 𝑥2 𝑝 ++𝑥𝑛 𝑝 𝑛 𝑝 ≤ √ 𝑥1 𝑞 + 𝑥2 𝑞 ++𝑥𝑛 𝑞 𝑛 𝑞 . Chứng minh. Cho 𝐴 = ( ∑𝑥𝑖 𝑝 𝑛 ) 1/𝑝 và 𝐵 = ( ∑𝑥𝑖 𝑞 𝑛 ) 1/𝑞 , 𝑖 = 1, , 𝑛 Khi đó 𝐵 𝐴 = ( 1 𝑛 [∑( 𝑥𝑖 𝐴 ) 𝑞 ]) 1/𝑞 = [ 1 𝑛 (∑[( 𝑥𝑖 𝐴 ) 𝑝 ] 𝑞/𝑝 )] 1/𝑞 . Áp duṇg bất đẳng thức Bernoulli 𝑦𝑟 ≥ 1 + 𝑟. (𝑦 − 1) với 𝑦 = ( 𝑥𝑖 𝐴 ) 𝑝 > 0 và 𝑟 = 𝑝 𝑞 ta thu đươc̣ 𝐵 𝐴 ≥ ( 1 𝑛 [∑(1 + 𝑞 𝑝 [( 𝑥𝑖 𝐴 ) 𝑝 − 1])]) 1 𝑞 = ( 1 𝑛 [𝑛 + 𝑞 𝑝 𝑛 − 𝑞 𝑝 𝑛]) 1 𝑞 = 1 Vậy √ 𝑥1 𝑝 +𝑥2 𝑝 ++𝑥𝑛 𝑝 𝑛 𝑝 ≤ √ 𝑥1 𝑞 +𝑥2 𝑞 ++𝑥𝑛 𝑞 𝑛 𝑞 .(□). Trung biǹh lũy thừa mũ 3 cho ba số và chỉ số nghèo Chỉ Số Nghèo (tên viết tắt bằng tiếng anh: HPI-1), là chỉ số phức hơp̣ đo mức sống trong một quốc gia đươc̣ đưa ra bởi liên hơp̣ quốc để đo lường mức độ đói nghèo của con người theo ba phương diện cơ bản: Tuổi thọ, sự hiểu biết và mức sống. Chỉ số được tính bằng công thức: 𝐻𝑃𝐼 − 1 = [(𝑃1 3 + 𝑃2 3 + 𝑃3 3)/3]1/3. Trong đó: P1: là tỉ lệ % người không sống được đến 40 tuổi. P2: là tỉ lệ người trưởng thành mù chữ. 48 P3: tỉ lệ % người không được tiếp cận với nguồn nước và trẻ em suy dinh dưỡng. Theo Báo Cáo Về Sự Phát Triển Con Người năm 2006, lũy thừa mũ 3 được dùng để cho thêm thông tin về mức độ nghèo ở các khu vực rơi vào tình trạng khẩn cấp. Trung bình lũy thừa, Piet Hein và siêu elip Cho các số dương 𝑎, 𝑏, 𝑝, xét các điểm (𝑥, 𝑦) trong mặt phẳng sao cho trung bình lũy thừa bậc p của |𝑥/𝑎| và|𝑦/𝑏| là không đổi, nghĩa là √(|𝑥/𝑎|𝑝 + |𝑦/𝑏|𝑝)/2 𝑝 = 𝑘. Với 𝑘 = 2−1/𝑝 ta có: |𝑥/𝑎|𝑝 + |𝑦/𝑏|𝑝 = 1 suy ra cho 𝑝 = 2 thì đồ thị của hàm số này là một đường elip nhưng với p > 2 thì nó được gọi là siêu elip (hoặc Lamé curve). Siêu elip được phổ biến rộng rãi bởi nhà toán học, nhà phát minh, nhà thơ người đan mạch và ông Piet Hein (1905- 1996), mặc dù chúng lần đầu tiên được nghiên cứu bởi nhà toán học người pháp có tên Gabriel Lamé (1795- 1870). Ông Hein đã sử dụng một siêu elip với 𝑝 = 2,5 và 𝑎 𝑏 = 6 5 trong một thiết kế năm 1959 dành cho cái bùng binh tại Sergels Torg, một quảng trường thành phố tại Stockholm, Thụy Điển và sân vận động Azteca được xây dựng cho sự kiện Olympie tại thành phố Mexico cũng có hình siêu elip. Thang Long University Library 49 2.9 Bất đẳng thức Schur Bất đẳng thức cổ điển của Issai Shur (1875-1941) phát biểu rằng: Cho ba số không âm 𝑥, 𝑦 , 𝑧 và số dương 𝑟, ta luôn có 𝑥𝑟(𝑥 − 𝑦)(𝑥 − 𝑧) + 𝑦𝑟(𝑦 − 𝑧)(𝑦 − 𝑥) + 𝑧𝑟(𝑧 − 𝑥)(𝑧 − 𝑦) ≥ 0. Đẳng thức xảy ra nếu 𝑥 = 𝑦 = 𝑧 hoăc̣ hai biến bằng nhau và biến thứ ba thì bằng 0. Với môṭ bất đẳng thức đối xứng đa ̃cho, chúng ta có thể giả sử rằng 𝑥 ≥ 𝑦 ≥ 𝑧 và bất đẳng thức trên đươc̣ viết laị như sau 𝑥𝑟(𝑥 − 𝑦)(𝑥 − 𝑧) + 𝑧𝑟(𝑥 − 𝑧)(𝑦 − 𝑧) ≥ 𝑦𝑟(𝑦 − 𝑧)(𝑥 − 𝑦). Ta hiểu mỗi số haṇg như môṭ thể tích của khối hôp̣. Vì 𝑥𝑟 ≥ 𝑦𝑟và 𝑥 − 𝑧 ≥ 𝑦 − 𝑧 nên ta có 𝑥𝑟(𝑥 − 𝑦)(𝑥 − 𝑧) ≥ 𝑦𝑟(𝑦 − 𝑧)(𝑥 − 𝑦) và do đó 𝑧𝑟(𝑥 − 𝑧)(𝑦 − 𝑧) ≥ 0, trong hình ve ̃2.18 ta thấy các hình hôp̣ có các thể tích là các số haṇg trong bất đẳng thức. Hiǹh 2.18 50 Bài toán 2.9 Cho 𝑎, 𝑏, 𝑐 biểu thi ̣đô ̣dài các caṇh của môṭ tam giác và 𝑠 là nửa chu vi. Chứng minh 𝑎𝑏𝑐𝑠 ≥ 𝑎3(𝑠 − 𝑎) + 𝑏3(𝑠 − 𝑏) + 𝑐3(𝑠 − 𝑐). (∗) Nhân 2 vào cả hai vế của bất đẳng thức (∗), ta có 2𝑎𝑏𝑐𝑠 ≥ 2𝑎3(𝑠 − 𝑎) + 2𝑏3(𝑠 − 𝑏) + 2𝑐3(𝑠 − 𝑐) ⇔ (𝑎 + 𝑏 + 𝑐)𝑎𝑏𝑐 ≥ 2𝑎3(𝑠 − 𝑎) + 2𝑏3(𝑠 − 𝑏) + 2𝑐3 ⇔𝑎2 𝑏𝑐 − 2𝑎3(𝑠 − 𝑎) + 𝑎𝑏2𝑐 − 2𝑏3(𝑠 − 𝑏) + 𝑎𝑏𝑐2 − 2𝑐3(𝑠 − 𝑐) ≥ 0. (∗∗) Bất đẳng thức (∗) tương đương với bất đẳng thức(∗∗), do đó để chứng minh (∗) ta đi chứng minh (∗∗). Thâṭ vâỵ, ta có 𝑎2𝑏𝑐 − 2𝑎3(𝑠 − 𝑎) = 𝑎2𝑏𝑐 − 𝑎3(𝑏 + 𝑐 − 𝑎) = 𝑎2(𝑎 − 𝑏)(𝑎 − 𝑐) ≥ 0 Tương tư ̣ta cũng có 𝑏2𝑎𝑐 − 2𝑏3(𝑠 − 𝑏) = 𝑏2(𝑏 − 𝑎)(𝑏 − 𝑐) ≥ 0. 𝑐2𝑎𝑏 − 2𝑐3(𝑠 − 𝑐) = 𝑐2(𝑐 − 𝑎)(𝑐 − 𝑏) ≥ 0. Do đó 𝑉𝑇 = 𝑎2(𝑎 − 𝑏)(𝑎 − 𝑐) + 𝑏2(𝑏 − 𝑎)(𝑏 − 𝑐) + 𝑐2(𝑐 − 𝑎)(𝑐 − 𝑏) ≥ 0. (⊡) (1). Nhận thấy (1) là môṭ trường hơp̣ đăc̣ biêṭ của bất đẳng thức Schur với 𝑟 = 2. Thang Long University Library 51 CHƯƠNG III MÔṬ SỐ BÀI TÂP̣ ÁP DUṆG 3.1 Bài tập áp dụng chương I Bài 3.1.1 Trong hình 120, cho một hình tứ giác không lồi (hình a) và một hình tứ giác đối xứng lệch (hình b). Chứng minh rằng. |𝐴𝐷| + |𝐷𝐶| < |𝐴𝐵| + |𝐵𝐶| và |𝐸𝐹| + |𝐺𝐻| < |𝐸𝐻| + |𝐹𝐺|. + Chứng minh. |𝐴𝐷| + |𝐷𝐶| < |𝐴𝐵| + |𝐵𝐶| Gọi 𝑀 = 𝐴𝐷 ∩ 𝐵𝐶 , khi đó |𝐴𝐵| + |𝐵𝑀| > |𝐴𝑀| (1) |𝐷𝑀| + |𝑀𝐶| > |𝐷𝐶| (2) Lấy (1) + (2) theo từng vế ta được |𝐴𝐵| + |𝐵𝑀| + |𝑀𝐶| + |𝐷𝑀| > |𝑀𝐴| + |𝐷𝐶| |𝐴𝐵| + |𝐵𝐶| + |𝐷𝑀| > |𝐴𝐷| + |𝐷𝑀| + |𝐷𝐶| |𝐴𝐵| + |𝐵𝐶| > |𝐴𝐷| + |𝐷𝐶|. (∗) + Chứng minh. |𝐸𝐹| + |𝐺𝐻| < |𝐸𝐻| + |𝐹𝐺| Gọi 𝑂 = 𝐹𝐺 ∩ 𝐸𝐻. Khi đó |𝑂𝐸| + |OF| > |𝐸𝐹| (3) A B C D F H G E B M D C A O F H G E 52 |𝑂𝐻| + |𝑂𝐺| > |𝐺𝐻| (4) Lấy (3), (4) theo từng vế ta được |𝐹𝑂| + |𝑂𝐸| + |𝑂𝐻| + |𝑂𝐺| > |𝐸𝐹| + |𝐻𝐺| |𝐸𝐹| + |𝐺𝐻| < |𝐸𝐻| + |𝐹𝐺|. (∗∗) Từ (∗) và (∗∗) ta có |𝐴𝐷| + |𝐷𝐶| < |𝐴𝐵| + |𝐵𝐶| và |𝐸𝐹| + |𝐺𝐻| < |𝐸𝐻| + |𝐹𝐺|. (□). Bài 3.1.2 Các bất đẳng thức cơ bản nào mà chúng ta có thể nhận được từ những hình sau. (a) (b) + Trong hình (a) có các bất đẳng thức cơ bản sau: Bất đẳng thức tam giác, bất đẳng thức AM- GM, bất đẳng thức giữa trung bình điều hòa và trung bình nhân. 1). 1 𝑎 + 1 𝑏 ≥ | 1 𝑎 − 1 𝑏 |. 2). 1 𝑎 + 1 𝑏 ≥ 2 √𝑎𝑏  2𝑎𝑏 𝑎+𝑏 ≤ √𝑎𝑏. 3). 1 𝑎 + 1 𝑏 + 2 √𝑎𝑏 ≥ | 1 𝑎 − 1 𝑏 |. + Trong hình (b) có bất đẳng thức cơ bản: 1 x + x ≥ 2. Chứng minh. Thang Long University Library 53 - TH1: Cho hình vuông ABCD có cạnh bằng 1. Lấy 𝐹 ∈ 𝐵𝐶 sao cho 𝐵𝐹 = 1 𝑥 . Gọi 𝐸 = 𝐴𝐹 ∩ 𝐷𝐶 sao cho 𝐷𝐸 = 𝑥, 𝑥 ≥ 1. Xét 2 tam giác đồng dạng CEF và DEA Có 𝐶𝐸 𝐷𝐸 = 𝐶𝐹 𝐷𝐴  𝐶𝐸 𝐷𝐸 = 𝐶𝐹 1  𝐶𝐸 = 𝐷𝐸. 𝐶𝐹. Hay 𝐶𝐸 ≥ 𝐶𝐹 x − 1 ≥ 1 − 1 x  1 x + x ≥ 2. - TH2: Cho hình vuông ABCD có cạnh bằng 1, lấy F∈CD sao cho DF = x, (0 < x  1). Gọi E = AF  BC và BE = 1 𝑥 . Xét 2 tam giác đồng dạng CFE và BAE Có 𝐶𝐸 𝐵𝐸 = 𝐶𝐹 𝐵𝐴  𝐶𝐸 𝐵𝐸 = 𝐶𝐹 1  𝐶𝐸 = 𝐵𝐸. 𝐶𝐹. Hay 𝐶𝐸 ≥ 𝐶𝐹  1 x − 1 ≥ 1 − x  1 x + x ≥ 2. Bài 3.1.3 cho 𝑎, 𝑏 là hai cạnh bên và 𝑐 là cạnh huyền của một tam giác vuông. Chứng minh bằng hình ảnh bất đẳng thức sau 𝑎 + 𝑏 ≤ 𝑐√2. Chứng minh. Xét hình vuông 𝐴𝐵𝐶𝐷 cạnh 𝑎 + 𝑏, trên các cạnh 𝐴𝐵, 𝐵𝐶, 𝐶𝐷, 𝐷𝐴 lần lượt lấy các điểm 𝑀, 𝑁, 𝑃, 𝑄 sao 54 cho tứ giác 𝑀𝑁𝑃𝑄 là hình vuông cạnh 𝑐 (như hình minh họa bên). Khi đó ta có đường chéo của hình vuông 𝑀𝑁𝑃𝑄 bằng 𝑐√2 ≥ a + b. (□) Bài 3.1.4 Cho 3 số dương 𝑎, 𝑏, 𝑐, chứng minh rằng √𝑎2 − 𝑎𝑏 + 𝑏2 + √𝑏2 − 𝑏𝑐 + 𝑐2 ≥ √𝑎2 + 𝑎𝑐 + 𝑐2. (1). Chứng minh. Cách 1 Biến đổi VT, ta có VT(1) = √(𝑏 – 𝑎 2 ) 2 + 3𝑎2 4 +√(𝑏 – 𝑐 2 ) 2 + 3𝑐2 4 . Đặt 𝑀 = (𝑏 − 𝑎 2 ; −𝑎√3 2 ), 𝑁 = (𝑏 − 𝑐 2 ; 𝑐√3 2 ). Khi đó: 𝑂𝑀 + 𝑂𝑁 ≥ 𝑀𝑁 √(𝑏– 𝑎 2 ) 2 + 3𝑎2 4 +√(𝑏– 𝑐 2 ) 2 + 3𝑐2 4 ≥ √( 𝑎 2 − 𝑐 2 ) 2 + ( 𝑎√3 2 − 𝑐√3 2 ) 2 √𝑎2 − 𝑎𝑏 + 𝑏2 + √𝑏2 − 𝑏𝑐 + 𝑐2 ≥ √𝑎2 + 𝑎𝑐 + 𝑐2.(□). Đẳng thức xảy ra khi và chỉ khi ba điểm 𝑂,𝑀,𝑁 thẳng hàng hay hai véc tơ 𝑂𝑀,⃗⃗⃗⃗⃗⃗⃗⃗ 𝑂𝑁⃗⃗⃗⃗⃗⃗ cùng phương, tức là: 𝑏− 𝑎 2 𝑏− 𝑐 2 = − 𝑎√3 2 𝑐√3 2  𝑏 = 𝑎𝑐 𝑎+𝑐 . Cách 2 Thang Long University Library 55 Xét tứ giác 𝑀𝑁𝑃𝑄 có 𝑀𝑁 = 𝑎,𝑀𝑃 = 𝑏,𝑀𝑄 = 𝑐 và các góc 𝑁𝑀𝑃 = 𝑄𝑀𝑃 = 600. Khi đó ta có : 𝑁𝑄2 = 𝑎2 + 𝑐2 − 2𝑎𝑐 cos 1200 = 𝑎2 + 𝑏2 + 𝑎𝑐. 𝑃𝑄2 = 𝑏2 + 𝑐2 − 2𝑏𝑐 cos 600 = 𝑏2 + 𝑐2 − 𝑏𝑐. 𝑁𝑃2 = 𝑎2 + 𝑏2 − 2𝑎𝑏 cos 600 = 𝑎2 + 𝑏2 − 𝑎𝑏. Áp dụng bất đẳng thức tam giác , ta có: 𝑁𝑃 + 𝑃𝑄 ≥ 𝑁𝑄. Đẳng thức xảy ra khi ba điểm 𝑁, 𝑃, 𝑄 thẳng hàng hay 𝑆𝑀𝑁𝑃 + 𝑆𝑀𝑄𝑃 = 𝑆𝑀𝑁𝑄  1 2 𝑎𝑏 sin 600 + 1 2 𝑏𝑐 sin 600 = 1 2 𝑎𝑐 sin 1200  ( 𝑎𝑏√3 4 ) + ( 𝑏𝑐√3 4 ) = 𝑎𝑐√3 4  𝑏 = 𝑎𝑐 𝑎+𝑐 . Vậy √𝑎2 − 𝑎𝑏 + 𝑏2 + √𝑏2 − 𝑏𝑐 + 𝑐2 ≥ √𝑎2 + 𝑎𝑐 + 𝑐2. (□) Bài 3.1.5 Sử dụng đồ thị của hàm số để chứng minh bất đẳng thức Bernouli (Johann Bernoulli, 1667 - 1748). Cho 𝑥 > 0 và 𝑟 > 1, 𝑥𝑟 − 1 ≥ 𝑟(𝑥 − 1). Chứng minh. Xét 2 hàm số 𝑓(𝑥) = 𝑥𝑟 − 1và 𝑔(𝑥) = 𝑟(𝑥 − 1). Ta có 𝑓′(𝑥) = 𝑟. 𝑥𝑟−1 > 0 với 𝑥 > 0, 𝑟 > 1 và 𝑔′(𝑥) = 𝑟 > 1. Từ đó hàm số 𝑓(𝑥) và 𝑔(𝑥) là các hàm đồng biến. Mặt khác 𝑓′(𝑥) = 𝑔′(𝑥) = 𝑟. Nên đồ thị của hàm số 𝑔(𝑥) là tiếp tuyến của đường cong 𝑓(𝑥) tại điểm (1; 0). Lại vì 𝑓′′(𝑥) = 𝑟(𝑟 − 1)𝑥𝑟−2 > 0 nên hàm số 𝑓(𝑥) là hàm lồi từ 56 A C y B x z y x z đó suy ra đồ thi ̣ của hàm số 𝑓(𝑥) nằm phía trên đồ thị hàm số 𝑔(𝑥). Đẳng thức xảy ra khi 𝑥 = 1. Vậy 𝑥𝑟 − 1 ≥ 𝑟(𝑥 − 1). (□). Bài 3.1.6 Chứng minh rằng nếu 𝑥 > 0 thì √𝑥 𝑥 ≤ √𝑒 𝑒 . (1) Chưng minh. Ta có √𝑥 𝑥 ≤ √𝑒 𝑒  𝑥 ≤ 𝑒 𝑥 𝑒. Từ đó dẫn đến chứng minh bài toán: Nếu 𝑥 > 0 thì 𝑥 ≤ 𝑒 𝑥 𝑒. Thật vậy, xét hai hàm số 𝑓(𝑥) = 𝑒 𝑥 𝑒 và 𝑔(𝑥) = 𝑥. Khi đó 𝑓′(𝑥) = 1 𝑒 𝑒 𝑥 𝑒 > 0 với 𝑥 > 0 và 𝑔′(𝑥) = 1 > 0. Suy ra 𝑓(𝑥) và 𝑔(𝑥) là các hàm đồng biến. Mặt khác 𝑓′(𝑥) = 𝑔′(𝑥) = 1 nên 𝑔(𝑥) = 𝑥 là tiếp xúc với đường cong 𝑓(𝑥) tại (𝑒; 𝑒). Vì 𝑓′′(𝑥) = 1 𝑒2 𝑒 𝑥 𝑒 > 0 nên 𝑓(𝑥) là hàm lồi trên (0; +∞). Hay đồ thị hàm số 𝑓(𝑥) nằm phía trên đồ thị hàm số 𝑔(𝑥). Từ đó 𝑒 𝑥 𝑒 ≥ x. Vậy √𝑒 𝑒 ≥ √𝑥 𝑥 . (□). Bài 3.1.7 cho 𝑎, 𝑏, 𝑐 biểu thị độ dài các cạnh của một tam giác và 𝑠 là nửa chu vi. Chứng minh các bất đẳng thức sau. a) 1 𝑠−𝑎 + 1 𝑠−𝑏 + 1 𝑠−𝑐 ≥ 9 𝑠 . (1) b) √𝑠 ≤ √𝑠 − 𝑎 + √𝑠 − 𝑏 + √𝑠 − 𝑐 ≤ √3𝑠 . (2) Chứng minh. Sử dụng đổi biến Ravi Đặt 𝑠 − 𝑎 = 𝑧, 𝑠 − 𝑏 = 𝑥, 𝑠 − 𝑐 = 𝑦. Thang Long University Library 57 suy ra 𝑠 = 𝑥 + 𝑦 + 𝑧. a) Bất đẳng thức (1)  1 𝑥 + 1 𝑦 + 1 𝑧 ≥ 9 x + y + z . (∗) Chứng minh bất đẳng thức (∗) Áp dụng bất đẳng thức giữa trung bình điều hòa và trung bình cộng cho ba số dương 𝑥, 𝑦, 𝑧, ta có 3 1 𝑥 + 1 𝑦 + 1 𝑧 ≤ 𝑥 + 𝑦 + 𝑧 3  1 𝑥 + 1 𝑦 + 1 𝑧 ≥ 9 x + y + z . Vậy 1 𝑠−𝑎 + 1 𝑠−𝑏 + 1 𝑠−𝑐 ≥ 9 𝑠 . (□). b) Bất đẳng thức (2) tương đương với bất đẳng thức √x + y + z ≤ √𝑥 + √𝑦 + √𝑧 ≤ √3(x + y + z) . (∗∗) - Chứng minh. √x + y + z ≤ √𝑥 + √𝑦 + √𝑧. Bình phương hai vế của bất đẳng thức trên, ta có 𝑥 + 𝑦 + 𝑧 ≤ 𝑥 + 𝑦 + 𝑧 + 2(√𝑥𝑦 + √𝑥𝑧 + √𝑦𝑧). (luôn đúng). (3) - Chứng minh. √𝑥 + √𝑦 + √𝑧 ≤ √3(x + y + z). Ta có (√𝑥 + √𝑦 + √𝑧) 2 = 𝑥 + 𝑦 + 𝑧 + 2(√𝑥𝑦 + √𝑥𝑧 + √𝑦𝑧) ≤ 𝑥 + 𝑦 + 𝑧 + 2( 𝑥 + 𝑦 2 + 𝑥 + 𝑧 2 + 𝑦 + 𝑧 2 ) = 3(𝑥 + 𝑦 + 𝑧). (4) Từ (3) và (4) suy ra √x + y + z ≤ √𝑥 + √𝑦 + √𝑧 ≤ √3(x + y + z). Vậy √𝑠 ≤ √𝑠 − 𝑎 + √𝑠 − 𝑏 + √𝑠 − 𝑐 ≤ √3𝑠. (□). 58 3.2 Bài tập áp dụng chương II Bài 3.2.1 Một dịch vụ chuyển phát nhanh thường hạn chế kích cỡ của một bưu kiên mà nó sẽ tiếp nhận, các gói bưu kiện không thể vượt quá 165 inches chiều dài cộng phần xung quanh, tức là dài + 2×rộng + 2×cao≤ 165. Tìm kích thước của gói bưu kiện được nhận với thể tích lớn nhất?. Lời giải. Gọi 𝑎, 𝑏, 𝑐 lần lượt biểu diễn cho chiều dài, rộng, cao của gói bưu kiện Khi đó bài toán trở thành: Tìm giá trị lớn nhất của thể tích hình hộp thỏa mãn : 𝑎 + 2𝑏 + 2𝑐 ≤ 165. Ký hiệu V là thể tích hình hộp suy ra 𝑉 = 𝑎𝑏𝑐. Đặt 𝑃 = 𝑎 + 2𝑏 + 2𝑐. Áp dụng AM-GM cho ba số dương 𝑎, 2𝑏, 2𝑐 ta có: 𝑃 ≥ 3√4𝑎𝑏𝑐 3 . Suy ra 𝑎𝑏𝑐 ≤ 𝑝3 27.4 ≤ 1653 4.27 . Khi đó 𝑚𝑎𝑥𝑉 = 𝑝3 27.4 khi { 𝑎 = 2𝑏 = 2𝑐 𝑎 + 2𝑏 + 2𝑐 = 165 ⇔ { 𝑎 = 55 𝑏 = 55/2 Vậy gói bưu kiện nhận được có thể tích lớn nhất khi chiều dài gấp đôi chiều rộng và chiều cao. Bài 3.2.2 Sử dụng phương pháp quy nạp và bất đẳng thức AM-GM cho hai số dương để chứng minh bất đẳng thức AM-GM cho 2k số dương, 𝑘 ≥ 1. Chứng minh. Thang Long University Library 59 Với 𝑘 = 1, theo bất đẳng thức AM-GM cho 𝑎, 𝑏 > 0 ta có 𝑎 + 𝑏 ≥ 2√𝑎𝑏. Giả sử bất đẳng thức cũng đúng với 𝑘 = 𝑝 ≥ 1, tức là ta có Với 𝑚 = 2𝑘 = 2𝑝: 𝑥1 + 𝑥2 +⋯+ 𝑥𝑚⏟ 𝑚 𝑠ố ≥ 𝑚 √𝑥1𝑥2𝑥𝑚 𝑚 . Ta cần chứng minh bất đẳng thức cũng đúng với 𝑘 = 𝑝 + 1. Hay chứng minh rằng với 2𝑘 = 2𝑝+1 = 2. 2𝑝 = 2𝑚, ta có 𝑥1 + 𝑥2 +⋯+ 𝑥𝑚⏟ + 𝑚 𝑠ố 𝑥𝑚+1 + 𝑥𝑚+2 +⋯+ 𝑥𝑚+𝑚⏟ 𝑚 𝑠ố ≥ 2𝑚 √𝑥1𝑥2𝑥2𝑚 2𝑚 . Thật vậy 𝑥1 + 𝑥2 +⋯+ 𝑥𝑚⏟ + 𝑚 𝑠ố 𝑥𝑚+1 + 𝑥𝑚+2 +⋯+ 𝑥𝑚+𝑚⏟ 𝑚 𝑠ố ≥ 𝑚 √𝑥1𝑥2𝑥𝑚 𝑚 +𝑚 √𝑥𝑚+1𝑥𝑚+2𝑥𝑚+𝑚 𝑚 ≥ 2𝑚 √𝑥1𝑥2𝑥2𝑚 2𝑚 . (□) Bài 3.2.3 Cho 𝐾 biểu thị diện tích của một tứ giác Q với độ dài các cạnh là 𝑎, 𝑏, 𝑐, 𝑑 theo thứ tự đó (tức là a và c đối nhau; b và d đối nhau). Thiết lập bất đẳng thức sau. a. 𝐾 ≤ 𝑎𝑏+𝑐𝑑 2 và 𝐾 ≤ 𝑎𝑑+𝑏𝑐 2 . b. 𝐾 ≤ 1 4 (𝑎 + 𝑐)(𝑏 + 𝑑). c. 𝐾 ≤ 1 16 (𝑎 + 𝑏 + 𝑐 + 𝑑)2. d. 𝐾 ≤ 1 4 (𝑎2 + 𝑏2 + 𝑐2 + 𝑑2). Chứng minh. Xét tứ giác ABCD ta có: 𝐴𝐵 = 𝑎, 𝐵𝐶 = 𝑏, 𝐶𝐷 = 𝑐, 𝐴𝐷 = 𝑑 và diện tích là 𝐾. 60 a) Ta có 𝐾 = 𝑆𝐴𝐵𝐶 + 𝑆𝐴𝐷𝐶 = 1 2 𝑎𝑏𝑠𝑖𝑛𝐵 + 1 2 𝑐𝑑 𝑠𝑖𝑛𝐷 ≤ 1 2 𝑎𝑏 + 1 2 𝑐𝑑 = 𝑎𝑏+𝑐𝑑 2 . (□) Đẳng thức xảy ra khi 𝐵 = 𝐷 = 900. Chứng minh tương tự ta có 𝐾 ≤ 𝑎𝑑+𝑏𝑐 2 b) Theo kết quả câu a) 𝐾 ≤ 𝑎𝑏+𝑐𝑑 2 , 𝐾 ≤ 𝑎𝑑+𝑏𝑐 2 Ta có 2𝐾 ≤ 𝑎𝑏 + 𝑐𝑑 2 + 𝑎𝑑 + 𝑏𝑐 2 = 𝑎𝑏 + 𝑏𝑐 2 + 𝑎𝑑 + 𝑐𝑑 2 = 1 2 (𝑎 + 𝑐)(𝑏 + 𝑑). Hay 𝐾 ≤ 1 4 (𝑎 + 𝑐)(𝑏 + 𝑑). Đẳng thức xảy ra khi 𝐴 = 𝐵 = 𝐶 = 𝐷 = 900 (tức Q là hình chữ nhật). (□) c) Theo câu b) 𝐾 ≤ 1 4 (𝑎 + 𝑐)(𝑏 + 𝑑). Áp dụng bất đẳng thức AM-GM cho hai số 𝑎 + 𝑐 và 𝑏 + 𝑑, ta có 𝐾 ≤ 1 4 (𝑎 + 𝑐)(𝑏 + 𝑑) ≤ 1 4 ( 𝑎 + 𝑏 + 𝑐 + 𝑑 2 ) 2 = 1 16 (𝑎 + 𝑏 + 𝑐 + 𝑑)2. Đẳng thức xảy ra khi { 𝑎 + 𝑐 = 𝑏 + 𝑑 𝐴 = 𝐵 = 𝐶 = 𝐷 = 900 (tức Q là hình vuông). Vậy 𝐾 ≤ 1 16 (𝑎 + 𝑏 + 𝑐 + 𝑑)2. (□) d) Áp dụng bất đẳng thức AM- RMS cho bốn số dương 𝑎, 𝑏, 𝑐, 𝑑, ta có 𝑎+𝑏+𝑐+𝑑 4 ≤ √ 𝑎2+𝑏2+𝑐2+𝑑2 4 . Hay 𝐾 = 1 16 (𝑎 + 𝑏 + 𝑐 + 𝑑)2 ≤ 1 4 (𝑎2 + 𝑏2 + 𝑐2 + 𝑑2). Thang Long University Library 61 Đẳng thức xảy ra khi 𝑎 = 𝑏 = 𝑐 = 𝑑 (tức Q là hình vuông). Vậy 𝐾 ≤ 1 4 (𝑎2 + 𝑏2 + 𝑐2 + 𝑑2). (□) Bài 3.2.4 Chứng minh rằng a. Mediant của hai phân số có cùng mẫu số là trung bình cộng. b. Mediant của hai phân số có cùng tử số là trung bình điều hòa. Chứng minh. a. Giả sử 2 phân số có mẫu số là: 𝑥 𝑎 ; 𝑦 𝑎 (𝑎 > 0). Khi đó mediant của 𝑥 𝑎 ; 𝑦 𝑎 là 𝑥+𝑦 2𝑎 = ( 𝑥 𝑎 )+( 𝑦 𝑎 ) 2 . (□) Vậy Mediant của hai phân số có cùng mẫu số là trung bình cộng. b. Giả sử hai phân số có cùng tử số là 𝑥 𝑎 ; 𝑥 𝑏 (𝑎 ≥ 𝑏 > 0). Khi đó mediant của 𝑥 𝑎 ; 𝑥 𝑏 là 2𝑥 𝑎+𝑏 = 2( 𝑥 𝑎 )( 𝑥 𝑏 ) (𝑥/𝑎)+(𝑥/𝑏) . (□) Vậy mediant của hai phân số có cùng tử số là trung bình điều hòa. Bài 3.2.5 Cho 𝑎, 𝑏, 𝑐 là các số dương. Chứng minh rằng 1 𝑎 + 𝑏 + 1 𝑏 + 𝑐 + 1 𝑐 + 𝑎 ≥ 9 2(𝑎 + 𝑏 + 𝑐) . Chưng minh. Cách 1 Áp dụng BĐT Chebyshev (2.2c), ta có: ( 1 𝑎 + 𝑏 + 1 𝑏 + 𝑐 + 1 𝑐 + 𝑎 ) ((𝑎 + 𝑏) + (𝑏 + 𝑐) + (𝑐 + 𝑎)) ≥ 9  2(𝑎 + 𝑏 + 𝑐) ( 1 𝑎+𝑏 + 1 𝑏+𝑐 + 1 𝑐+𝑎 ) ≥ 9. Hay 1 𝑎+𝑏 + 1 𝑏+𝑐 + 1 𝑐+𝑎 ≥ 9 2(𝑎+𝑏+𝑐) . Đẳng thức xảy ra khi 𝑎 = 𝑏 = 𝑐. Vậy 1 𝑎+𝑏 + 1 𝑏+𝑐 + 1 𝑐+𝑎 ≥ 9 2(𝑎+𝑏+𝑐) . (□) Cách 2 62 Áp dụng bất đẳng thức giữa trung bình điều hòa và trung bình cộng cho ba số 1 𝑎+𝑏 , 1 𝑏+𝑐 , 1 𝑐+𝑎 , ta có 1 (𝑎+𝑏)+(𝑏+𝑐)+(𝑐+𝑎) 3 ≤ 1 𝑎+𝑏 + 1 𝑏+𝑐 + 1 𝑐+𝑎 3  3 2(𝑎+𝑏+𝑐) ≤ 1 𝑎+𝑏 + 1 𝑏+𝑐 + 1 𝑐+𝑎 3  1 𝑎+𝑏 + 1 𝑏+𝑐 + 1 𝑐+𝑎 ≥ 9 2(𝑎+𝑏+𝑐) . (□) Bài 3.2.6 Cho 𝑎 ≥ 𝑏 ≥ 𝑐 ≥ 0 và 𝑎 + 𝑏 + 𝑐 ≤ 1. Chứng minh bằng hình vẽ 𝑎2 + 3𝑏2 + 5𝑐2 ≤ 1 Chứng minh. Ta có: 𝑎 + 𝑏 + 𝑐 ≤ 1 suy ra (𝑎 + 𝑏 + 𝑐)2 ≤ 1. Để chứng minh 𝑎2 + 3𝑏2 + 5𝑐2 ≤ 1 Cần chứng minh 𝑎2 + 3𝑏2 + 5𝑐2 ≤ (𝑎 + 𝑏 + 𝑐)2 Từ hình vẽ ta thấy: Các hình vuông có diện tích 𝑎2 + 3𝑏2 + 5𝑐2 thì lắp vừa khít bên trong một hình vuông có diện tích (𝑎 + 𝑏 + 𝑐)2. Do đó 𝑎2 + 3𝑏2 + 5𝑐2 ≤ (𝑎 + 𝑏 + 𝑐)2 ≤ 1. (□). Bài 3.2.7 Chứng minh rằng bất đẳng thức Schur dẫn tới bất đẳng thức Padoa. Chứng minh. Ta có với 𝑟 = 1 bất đẳng thức Schur có dạng 𝑥(𝑥 − 𝑦)(𝑥 − 𝑧) + 𝑦(𝑦 − 𝑧)(𝑦 − 𝑥) + 𝑧(𝑧 − 𝑥)(𝑧 − 𝑦) ≥ 0 𝑥3 + 𝑦3 + 𝑧3 − 𝑥2𝑧 − 𝑥2𝑦 + 3𝑥𝑦𝑧 − 𝑥𝑦2 − 𝑦2𝑧 − 𝑧2𝑦 − 𝑧2𝑥 ≥ 0 Thang Long University Library 63  𝑥𝑦𝑧 ≥ (2𝑥2𝑦 + 2𝑥𝑦2 − 2𝑥𝑦𝑧) + (𝑥𝑧2 + 𝑦𝑧2 − 𝑧3) + (𝑦2𝑧 − 𝑦2𝑥 − 𝑦3) + (𝑥2𝑧 − 𝑥2𝑦 − 𝑥3) ≥ 2𝑥𝑦(𝑥 + 𝑦 − 𝑧) + 𝑧2(𝑥 + 𝑦 − 𝑧) − 𝑦2(𝑥 + 𝑦 − 𝑧) − 𝑥2(𝑥 + 𝑦 − 𝑧) ≥ (𝑥 + 𝑦 − 𝑧)(2𝑥𝑦 + 𝑧2 − 𝑦2 − 𝑥2) ≥ (𝑥 + 𝑦 − 𝑧)((𝑥𝑦 + 𝑦𝑧 − 𝑦2) + (𝑥𝑦 − 𝑥2 − 𝑥𝑧) + (𝑧2 − 𝑦𝑧 + 𝑥𝑧)) ≥ (𝑥 + 𝑦 − 𝑧)(𝑥 − 𝑦 + 𝑧)(−𝑥 + 𝑦 + 𝑧) Vậy với 𝑟 = 1 thì bất đẳng thức Schur dẫn tới bất đẳng thức Padoa. (□) Bài 3.2.8 Sử dụng bất đẳng thức Chebysev để thiết lập bất đẳng thức Mengoli. Chứng minh. Áp dụng bất đẳng thức Chebysev (2.2c), ta có (𝑥 + 1 + 𝑥 + 𝑥 − 1) ( 1 𝑥 + 1 + 1 𝑥 + 1 𝑥 − 1 ) ≥ 9 ⇔3𝑥 ( 1 𝑥 + 1 + 1 𝑥 + 1 𝑥 − 1 ) ≥ 9 ⇔( 1 𝑥 + 1 + 1 𝑥 + 1 𝑥 − 1 ) ≥ 3 𝑥 Vậy bất đẳng thức Mengoli đã được thiết lập. (□) Bài 3.2.9 Chứng minh rằng trong tất cả các hình hộp chữ nhật thì hình vuông có diện tích mặt ngoài nhỏ nhất. Chứng minh. Ký hiệu 𝑆 là diện tích mặt ngoài; 𝑎, 𝑏, 𝑐 lần lượt là chiều dài, rộng và chiều cao còn V là thể tích của hình hộp. Khi đó 𝑆 = 2(𝑎𝑏 + 𝑏𝑐 + 𝑐𝑎) ≥ 6√𝑎2𝑏2𝑐2 3 = 6√𝑉2 3 ⇔ 𝑆 ≥ 6√𝑉2 3 Đẳng thức xảy ra khi và chỉ khi 𝑎 = 𝑏 = 𝑐. Do đó nếu V cố định thì hình vuông có diện tích mặt ngoài là nhỏ nhất. (□) 64 Bài 3.2.10 Dùng hình ảnh, chứng minh rẳng với mọi 𝑥 ≥ 𝑦 ≥ 0, ta luôn có √3𝑦2 + 𝑥2 ≤ 𝑥 + 𝑦 ≤ √3𝑥2 + 𝑦2 (𝑎) Chưng minh. Ta có: √3𝑦2 + 𝑥2 ≤ 𝑥 + 𝑦 ≤ √3𝑥2 + 𝑦2 ⇔3𝑦2 + 𝑥2 ≤ (𝑥 + 𝑦)2 ≤ 3𝑥2 + 𝑦2 (𝑏) Để chứng minh (b), ta có hình sau. Từ hình vẽ trên ta thấy các hình vuông có diện tích 3𝑦2 + 𝑥2 thì lắp vừa khít bên trong hình vuông cạnh (𝑥 + 𝑦) (1) Còn hình vuông cạnh (𝑥 + 𝑦) thì lắp vừa khít bên trong hình được tạo bởi các hình vuông có diện tích là 3𝑥2 + 𝑦2 (2) Từ (1) và (2) ta có: 3𝑦2 + 𝑥2 ≤ (𝑥 + 𝑦)2 ≤ 3𝑥2 + 𝑦2 Vậy √3𝑦2 + 𝑥2 ≤ 𝑥 + 𝑦 ≤ √3𝑥2 + 𝑦2 (□) Bài 3.2.11 Cho 𝑎 ≥ 𝑏 ≥ 𝑐 là ba cạnh của tam giác. Chứng minh rằng 𝑎2 + 𝑏2 + 𝑐2 ≤ (𝑎 + 𝑏)(𝑏 + 𝑐) Chứng minh. Thang Long University Library 65 Nhìn vào hình vẽ ta thấy các hình vuông có các cạnh cạnh lần lượt là a,b,c lắp vừa khít bên trong hình chữ nhật có diện tích là (𝑎 + 𝑏)(𝑏 + 𝑐) Suy ra diện tích của các hình vuông (𝑎2 + 𝑏2 + 𝑐2) nhỏ hơn hoặc bằng diện tích của hình chữ nhật (𝑎 + 𝑏)(𝑏 + 𝑐) Vậy 𝑎2 + 𝑏2 + 𝑐2 ≤ (𝑎 + 𝑏)(𝑏 + 𝑐). (□). Bài 3.2.12. Chứng minh công thức tổng quát của tổng Mediant trong trường hợp tổng quát sau: Nếu 𝑎1; 𝑎2; , , 𝑎𝑛 là các số thực và 𝑏1; 𝑏2; , 𝑏𝑛 là các số thực dương thì min 1≤𝑖≤𝑛 𝑎𝑖 𝑏𝑖 ≤ 𝑎1 + 𝑎2 +⋯+ 𝑎𝑛 𝑏1 + 𝑏2 +⋯+ 𝑏𝑛 ≤ max 1≤𝑖≤𝑛 𝑎𝑖 𝑏𝑖 Chứng minh. Không mất tính tổng quát, ta giả sử: 𝑎1 𝑏1 ≤ 𝑎2 𝑏2 ≤ ⋯ ≤ 𝑎𝑛 𝑏𝑛 Khi đó min 1≤𝑖≤𝑛 𝑎𝑖 𝑏𝑖 = 𝑎1 𝑏1 , max 1≤𝑖≤𝑛 𝑎𝑖 𝑏𝑖 = 𝑎𝑛 𝑏𝑛 Ta có mediant của 𝑎1 𝑏1 , 𝑎2 𝑏2 là 𝑎1+𝑎2 𝑏1+𝑏2 (𝑛 = 2) Tương tự, ta cũng có Mediant của 𝑎1+𝑎2 𝑏1+𝑏2 và 𝑎3 𝑏3 là 𝑎1+𝑎2+𝑎3 𝑏1+𝑏2+𝑏3 Mediant của 𝑎1+𝑎2+⋯+𝑎𝑛−1 𝑏1+𝑏2+⋯+𝑏𝑛−1 và 𝑎𝑛 𝑏𝑛 là 𝑎1+𝑎2+⋯+𝑎𝑛 𝑏1+𝑏2+⋯+𝑏𝑛 66 Suy ra 𝑎1 𝑏1 ≤ 𝑎1 + 𝑎2 +⋯+ 𝑎𝑛 𝑏1 + 𝑏2 +⋯+ 𝑏𝑛 ≤ 𝑎𝑛 𝑏𝑛 Vậy ta luôn có: min 1≤𝑖≤𝑛 𝑎𝑖 𝑏𝑖 ≤ 𝑎1+𝑎2+⋯+𝑎𝑛 𝑏1+𝑏2+⋯+𝑏𝑛 ≤ max 1≤𝑖≤𝑛 𝑎𝑖 𝑏𝑖 . (□). Bài 3.2.13 Cặp hình dưới đây minh họa một bất đẳng thức cơ bản. Đó là bất đẳng thức nào? a) b) Lời giải. Từ hình (b) ta có: 1 2 (√𝑎) 2 + 1 2 (𝑏)2 ≥ √𝑎√𝑏. Hay 𝑎+𝑏 2 ≥ √𝑎𝑏. Đây là bất đẳng thức cơ bản AM-GM. Bài 3.2.14 Nếu 𝑎, 𝑏, 𝑐 là ba cạnh của một tam giác vuông có cạnh huyền là 𝑐 thì 𝑎𝑐 + 𝑏2 ≤ 5𝑐2/4. Chứng minh. Vì a, b, c là ba cạnh của một tam giác vuông có cạnh huyền là c nên ta có 𝑎2 + 𝑏2 = 𝑐2⇔𝑐2 − 𝑎2 = 𝑏2. Từ đó bất đẳng thức 𝑎𝑐 + 𝑏2 ≤ 5𝑐2/4⇔𝑎𝑐 − 𝑎2 ≤ 𝑐2/4 Cần chứng minh bất đẳng thức 𝑎𝑐 − 𝑎2 ≤ 𝑐2/4. Thật vậy: 𝑎𝑐 = 2𝑎(𝑐/2) ≤ 𝑎2 + 𝑐2/4⇔𝑎𝑐 − 𝑎2 ≤ 𝑐2/4. Vậy 𝑎𝑐 + 𝑏2 ≤ 5𝑐2/4. (□) Thang Long University Library 67 Bài 3.2.15 Cho 𝑥, 𝑦 ∈ [0; 1] Chứng minh bằng hình vẽ : √𝑥𝑦 + √(1 − 𝑥)(1 − 𝑦) ≤ 1. Chứng minh. Từ hình vẽ ta thấy 1 − √𝑥𝑦 ≥ √(1 − 𝑥)(1 − 𝑦) Do đó √𝑥𝑦 + √(1 − 𝑥)(1 − 𝑦) ≤ 1 (□). 68 KẾT LUÂṆ Trong luâṇ văn này tác giả đã đọc, nghiên cứu và viết lại một số các kết quả sau. 1. Thiết lập được bất đẳng thức AM-GM cho hai số dương và một số bất đẳng thức trung bình khác bằng cách sử dụng một trong các phương pháp như: So sánh Pythagore, bất đẳng thức tam giác, nguyên lý trắc địa và so sánh đồ thị các hàm số. 2. Thiết lập được bất đẳng thức AM –GM từ ba số đến n số dương, bất đẳng thức Guha, bất đẳng thức Chebyshev, bất đẳng thức Schur và một số bất đẳng thức khác bằng cách sử dụng phương pháp nguyên lý bao hàm. 3. Vận dụng được các phương pháp nói trên vào việc giải các bài toán chứng minh một số bất đẳng thức cơ bản bằng phương pháp hình học và một số bài toán có liên quan. Thang Long University Library 69 TÀI LIÊỤ THAM KHẢO [1] Alsina, Claudi, and Roger B. Nelsen. When Less is More: Visualizing Basic Inequalities. No. 36. MAA, 2009. [2] Alsina, C, “The arithmetic mean- geometric mean inequality for three positive numbers”, Mathematics Magazine, 73 (2000), p. 97. [3] Arnol'd, V. I, Yesterday and Long Ago, Springer, Berlin, 2006. [4] D�̈�rrie, 100 Great Problems of Elementary Mathematics, Dover, New York, 1965. [5] Ferréol, R, personal communication (2006). [6] Guba, S. G, "Zadaca 1797”, Mat. V �̌�kole (1977), p. 80. [7] Kazarinoff, N. D, Geometric Inequalities, Mathematical Association of America, Washington, 1961. [8] Maor, E, Trigonometric Delights, Princeton University Press, Princeton, 1998. [9] Nelsen, R. B, "The harmonic mean-geometric mean-arithmetic mean- root mean square inequality," Mathematics Magazine, 60 (1987), p. 158. [10] Nelsen, R, B, “The sum of a positive number and its reciprocal is at leats two (four proofs)”, Mathematics Magazine, 67 (1994b), p. 374. [11] Nesbitt, A. M, "Problem 1514”, Educational Times, 2 (1903), pp. 37- 38. [12] Niven, I, Maxima and Minima Without Calculus, Mathematical Association of America, Washington, 1981. [13] Padoa, A, "Una questione di minimo," Periodico di Matematiche, 4 (1925), pp. 80-85. [14] Shklarsky, O, N. N. Chentzov and I. M. Yaglon, The USSR Olympiad Problem Book, W. H. Freeman and Co., San Francisco, 1962. [15] Voicu, I, "Problem 18666", Gaz. Mat., 86 (1981), p. 112. CỘNG HÒA XÃ HỘI CHỦ NGHĨA VIỆT NAM Độc lập – Tự do – Hạnh phúc GIẤY XÁC NHẬN CHỈNH SỬA LUẬN VĂN THẠC SĨ Họ và tên tác giả luận văn: Nguyễn Thị Minh Trang. Đề tài luận văn: CHỨNG MINH MỘT SỐ BẤT ĐẲNG THỨC CƠ BẢN BẰNG PHƯƠNG PHÁP HÌNH HỌC. Chuyên ngành: Toán và Thống Kê Mã Học viên: C00271 Cơ sở đào tạo: Trường Đại học Thăng Long Căn cứ vào biên bản cuộc họp Hội đồng chấm luận văn thạc sĩ ngày 07/06/2016 tại Trường Đại học Thăng Long và các nhận xét, góp ý cụ thể của các thành viên hội đồng, tác giả luận văn đã thực hiện các chỉnh sửa sau: 1. Chỉnh lại một số câu văn trong trình bày. 2. Chỉnh lại các dấu chấm câu, các lỗi trình bày văn bản. Hà nội, ngày 10 tháng 07 năm 2016 Xác nhận của giáo viên hướng dẫn Tác giả luận văn Nguyễn Thị Minh Trang Xác nhận của Chủ tịch Hội đồng chấm luận văn Thang Long University Library

Các file đính kèm theo tài liệu này:

  • pdfc00271_4053_1427.pdf
Luận văn liên quan